Difference between revisions of "2009 AMC 12B Problems/Problem 23"

m (Solution 2 (Same idea))
m (Solution 2 (Same idea))
Line 38: Line 38:
 
The solution proposed above is good, but there is a more straightforward method. First, turn <math>\frac34 + \frac34i</math> into polar form as <math>\frac{3\sqrt{2}}{4}e^{\frac{\pi}{4}i}</math>. Restated using geometric probabilities, we are trying to find the portion of a square enlarged by a factor of <math>\frac{3\sqrt{2}}{4}</math> and rotated <math>45</math> degrees that lies within the original square. This skips all the absolute values required before. Finish with the symmetry method stated above.
 
The solution proposed above is good, but there is a more straightforward method. First, turn <math>\frac34 + \frac34i</math> into polar form as <math>\frac{3\sqrt{2}}{4}e^{\frac{\pi}{4}i}</math>. Restated using geometric probabilities, we are trying to find the portion of a square enlarged by a factor of <math>\frac{3\sqrt{2}}{4}</math> and rotated <math>45</math> degrees that lies within the original square. This skips all the absolute values required before. Finish with the symmetry method stated above.
  
-Rowechen
+
-asdf334
  
 
== See Also ==
 
== See Also ==

Revision as of 18:31, 11 May 2019

Problem

A region $S$ in the complex plane is defined by \[S = \{x + iy: - 1\le x\le1, - 1\le y\le1\}.\] A complex number $z = x + iy$ is chosen uniformly at random from $S$. What is the probability that $\left(\frac34 + \frac34i\right)z$ is also in $S$?

$\textbf{(A)}\ \frac12\qquad \textbf{(B)}\ \frac23\qquad \textbf{(C)}\ \frac34\qquad \textbf{(D)}\ \frac79\qquad \textbf{(E)}\ \frac78$

Solution

We can directly compute $\left(\frac34 + \frac34i\right)z = \left(\frac34 + \frac34i\right)(x + iy) = \frac{3(x-y)}4 + \frac{3(x+y)}4 \cdot i$.

This number is in $S$ if and only if $-1 \leq \frac{3(x-y)}4 \leq 1$ and at the same time $-1 \leq \frac{3(x+y)}4 \leq 1$. This simplifies to $|x-y|\leq\frac 43$ and $|x+y|\leq\frac 43$.

Let $T = \{ x + iy : |x-y|\leq\frac 43 ~\land~ |x+y|\leq\frac 43 \}$, and let $[X]$ denote the area of the region $X$. Then obviously the probability we seek is $\frac {[S\cap T]}{[S]} = \frac{[S\cap T]}4$. All we need to do is to compute the area of the intersection of $S$ and $T$. It is easiest to do this graphically:

[asy] unitsize(2cm); defaultpen(0.8); path s = (-1,-1) -- (-1,1) -- (1,1) -- (1,-1) -- cycle; path t = (4/3,0) -- (0,4/3) -- (-4/3,0) -- (0,-4/3) -- cycle; path s_cap_t = (1/3,1) -- (1,1/3) -- (1,-1/3) -- (1/3,-1) -- (-1/3,-1) -- (-1,-1/3) -- (-1,1/3) -- (-1/3,1) -- cycle; filldraw(s, lightred, black); filldraw(t, lightgreen, black); filldraw(s_cap_t, lightyellow, black); draw( (-5/3,0) -- (5/3,0), dashed ); draw( (0,-5/3) -- (0,5/3), dashed ); [/asy]

Coordinate axes are dashed, $S$ is shown in red, $T$ in green and their intersection is yellow. The intersections of the boundary of $S$ and $T$ are obviously at $(\pm 1,\pm 1/3)$ and at $(\pm 1/3,\pm 1)$.

Hence each of the four red triangles is an isosceles right triangle with legs long $\frac 23$, and hence the area of a single red triangle is $\frac 12 \cdot \left( \frac 23 \right)^2 = \frac 29$. Then the area of all four is $\frac 89$, and therefore the area of $S\cap T$ is $4 - \frac 89$. Then the probability we seek is $\frac{ [S\cap T]}4 = \frac{ 4 - \frac 89 }4 = 1 - \frac 29 = \boxed{\frac 79}$.

(Alternately, when we got to the point that we know that a single red triangle is $\frac 29$, we can directly note that the picture is symmetric, hence we can just consider the first quadrant and there the probability is $1 - \frac 29 = \frac 79$. This saves us the work of first multiplying and then dividing by $4$.)

Solution 2 (Same idea)

The solution proposed above is good, but there is a more straightforward method. First, turn $\frac34 + \frac34i$ into polar form as $\frac{3\sqrt{2}}{4}e^{\frac{\pi}{4}i}$. Restated using geometric probabilities, we are trying to find the portion of a square enlarged by a factor of $\frac{3\sqrt{2}}{4}$ and rotated $45$ degrees that lies within the original square. This skips all the absolute values required before. Finish with the symmetry method stated above.

-asdf334

See Also

2009 AMC 12B (ProblemsAnswer KeyResources)
Preceded by
Problem 22
Followed by
Problem 24
1 2 3 4 5 6 7 8 9 10 11 12 13 14 15 16 17 18 19 20 21 22 23 24 25
All AMC 12 Problems and Solutions

The problems on this page are copyrighted by the Mathematical Association of America's American Mathematics Competitions. AMC logo.png